Merge remote branch 'public/master'
[course.git] / latex / problems / Serway_and_Jewett_4 / problem19.55.tex
1 \begin{problem*}{19.55}
2 Four identical point charges ($q = +10.0\U{$\mu$C}$) are located on
3 the corners of a rectangle as shown in Figure P19.55.  The dimensions
4 of the rectangle are $L = 60.0\U{cm}$ and $W = 15.0\U{cm}$.  Calculate
5 the magnitude and direction of the resultant electric force exerted on
6 the charge at the lower left corner by the other three charges.
7 \end{problem*} % problem 19.55
8
9 \empaddtoprelude{
10   numeric a;
11   pair A, B, C, D;
12   a := 1cm;
13   A := (0, a); % q1, labeled CCW from upper left
14   B := origin; % q2
15   C := (4a, 0); % q3
16   D := (4a, a); % q4
17   def drawE = 
18     label.rt("W", draw_length(C, D, 8pt));
19     label.bot("L", draw_length(B, C, 8pt));
20     labeloffset := 5pt;
21     draw_pcharge(A, 3pt);
22     label.bot(btex $q_1$ etex, A);
23     draw_pcharge(B, 3pt);
24     label.top(btex $q_2$ etex, B);
25     draw_pcharge(C, 3pt);
26     label.top(btex $q_3$ etex, C);
27     draw_pcharge(D, 3pt);
28     label.bot(btex $q_4$ etex, D);
29   enddef;
30 }
31 \begin{nosolution}
32 \begin{center}
33 \begin{empfile}[3p]
34 \begin{emp}(0cm,0cm)
35   drawE;
36 \end{emp}
37 \end{empfile}
38 \end{center}
39 \end{nosolution}
40
41 \begin{solution}
42 \begin{center}
43 \begin{empfile}[3]
44 \begin{emp}(0cm,0cm)
45   label.rt(btex $F_{12}$ etex, draw_force(A, B, 2cm));
46   label.lft(btex $F_{32}$ etex, draw_force(C, B, .5cm));
47   label.bot(btex $F_{42}$ etex, draw_force(D, B, .48cm));
48   draw A--(D+(a/2,0)) dashed evenly;
49   draw A--B dashed evenly;
50   draw B--D dashed evenly;
51   label.top(btex $\theta$ etex, draw_lout_angle(D+(1,0),D,B, a/3));
52   draw_right_angle(B, A, D, a/3);
53   draw_ijhats((5.5*a, a/3), 0, a/3);
54   drawE;
55 \end{emp}
56 \end{empfile}
57 \begin{center}
58 \begin{empfile}[3]
59 \begin{emp}(0cm,0cm)
60   label.rt(btex $F_{12}$ etex, draw_force(A, B, 2cm));
61   label.lft(btex $F_{32}$ etex, draw_force(C, B, .5cm));
62   label.bot(btex $F_{42}$ etex, draw_force(D, B, .48cm));
63   draw A--(D+(a/2,0)) dashed evenly;
64   draw A--B dashed evenly;
65   draw B--D dashed evenly;
66   label.top(btex $\theta$ etex, draw_lout_angle(D+(1,0),D,B, a/3));
67   draw_right_angle(B, A, D, a/3);
68   draw_ijhats((5.5*a, a/3), 0, a/3);
69   drawE;
70 \end{emp}
71 \end{empfile}
72 The unit vector \rhat\ diagonally across from the upper right is given by
73 \begin{align}
74  \rhat &= \cos\theta \ihat + \sin\theta \jhat \\
75  \theta &= \arctan{W/L} + 180\dg = 194\dg \\
76  \cos\theta &= -0.970 \\
77  \sin\theta &= -0.243
78 \end{align}
79 So the electric field in the lower left corner is given by
80 \begin{align}
81  \vect{E} &= k_e \sum_i \frac{q_i}{r_i^2}\rhat_i
82            = k_e \left(\frac{q}{L^2}(-\ihat)
83                        + \frac{q}{(L^2 + W^2)}(\cos\theta\ihat + \sin\theta\jhat)
84                        + \frac{q}{W^2}(-\jhat) \right) \\
85           &= -k_e q \left[
86                \left(\frac{1}{L^2} - \frac{\cos\theta}{L^2+W^2}\right)\ihat
87                + \left(\frac{1}{W^2} - \frac{\sin\theta}{L^2+W^2}\right)\jhat
88                     \right]
89 \end{align}
90
91 So the magnitude of \vect{E} is given by
92 \begin{equation}
93  E = k_e q \sqrt{ \left(L^{-2} - \frac{\cos\theta}{L^2+W^2}\right)^2
94                 + \left(W^{-2} - \frac{\sin\theta}{L^2+W^2}\right)^2 }
95   = 4.08\E{6}\U{N/C}
96 \end{equation}
97 (Remembering to convert $L$ and $W$ to meters.)  And the direction
98 $\phi$ (measured counter clockwise from \ihat) of \vect{E} is given by
99 \begin{equation}
100  \phi = \arctan\left(\frac{-W^{-2}+\frac{\sin\theta}{L^2+W^2}}{-L^{-2}+\frac{\cos\theta}{L^2+W^2}}\right) + 180\dg
101         = \ans{263\dg}
102 \end{equation}
103 Where the $+180\dg$ is because the tangent has a period of $180\dg$,
104 and the angle we want is in the backside $180\dg$.
105
106 $\vect{F} = q \vect{E}$ so the direction of \vect{F} is the same as
107 the direction of \vect{E}.  The magnitude of \vect{F} is given by
108 \begin{equation}
109  F = 10.0\E{-6}\U{C} \cdot 4.08\E{6}\U{N/C} = \ans{40.8\U{N}}
110 \end{equation}
111 \end{solution}